Diễn Đàn MathScopeDiễn Đàn MathScope
  Diễn Đàn MathScope
Ghi Danh Hỏi/Ðáp Community Lịch

Go Back   Diễn Đàn MathScope > Sơ Cấp > Tài Liệu > Đề Thi > Đề Thi HSG Cấp Tỉnh ở Việt Nam

News & Announcements

Ngoài một số quy định đã được nêu trong phần Quy định của Ghi Danh , mọi người tranh thủ bỏ ra 5 phút để đọc thêm một số Quy định sau để khỏi bị treo nick ở MathScope nhé !

* Nội quy MathScope.Org

* Một số quy định chung !

* Quy định về việc viết bài trong diễn đàn MathScope

* Nếu bạn muốn gia nhập đội ngũ BQT thì vui lòng tham gia tại đây

* Những câu hỏi thường gặp

* Về việc viết bài trong Box Đại học và Sau đại học


Trả lời Gởi Ðề Tài Mới
 
Ðiều Chỉnh Xếp Bài
Old 19-10-2017, 12:47 AM   #16
312cr9
+Thành Viên+
 
Tham gia ngày: Aug 2014
Bài gởi: 10
Thanks: 3
Thanked 2 Times in 2 Posts
Trích:
Nguyên văn bởi MATHSCOPE View Post

$\boxed{8}$ [Hoà Bình] Tìm tất cả các cặp số nguyên dương $(a;\,b)$ sao cho$\left(a^2+b\right)\left(a+b^2\right)$ là một luỹ thừa của 2.
Nếu $a;\,b$ thoả yêu cầu thì sẽ phải tồn tại $m;\,n\in\mathbb Z^+$ thoả
\[\begin{array}{l}
{a^2} + b = {2^m}\\
{b^2} + a = {2^n}
\end{array}\]
Xét hai trường hợp sau
  1. Nếu $a\ne b$, ta giả sử $a>b$ và thấy
    \[{2^m} - {2^n} = \left( {a - b} \right)\left( {a + b - 1} \right)\]
    Như vậy, $m>n$ ta viết $m=n+k$ với $k\in\mathbb Z^+$ để có
    \[\left( {a - b} \right)\left( {a + b - 1} \right) = {2^n}\left( {{2^k} - 1} \right)\]
    Giờ để ý rằng $a;\,b$ có cùng tính chẵn lẻ cho nên $a+b-1$ lẻ tức là
    \[a-b\;\vdots\;2^n\]
    Điều này dẫn đến $a-b\ge 2^n=a+b^2$ tức $b^2+b\le 0$, và điều đó không thể xảy ra.
  2. Nếu $a=b$, thế thì $a(a+1)=2^m$. Tức $a$ và $a+1$ không được có ước nguyên tố nào ngoài 2. Mà $\gcd(a;\,a+1)=1$ và $a+1>1$ nên chỉ có thể $a=1$. Với $a=b=1$, ta thử thấy thoả.
Tóm lại $a=b=1$.
[RIGHT][I][B]Nguồn: MathScope.ORG[/B][/I][/RIGHT]
 
312cr9 is offline   Trả Lời Với Trích Dẫn
Old 19-10-2017, 04:41 PM   #17
Thụy An
+Thành Viên+

 
Tham gia ngày: Oct 2017
Bài gởi: 93
Thanks: 1
Thanked 68 Times in 45 Posts
Trích:
Nguyên văn bởi MATHSCOPE View Post
$\boxed{31}$ [Hoà Bình] Tìm các hàm số $f:\, \mathbb{R}\rightarrow \mathbb{R}$ thoả mãn
\[f\left( x \right) + f\left( y \right) + f\left( {xy} \right) = f\left( {x + y} \right) + f\left( x \right)f\left( y \right)\quad\forall\,x;\,y\in\mathbb R.\]
Gọi $P(x;\,y)$ là mệnh đề: $f\left( x \right) + f\left( y \right) + f\left( {xy} \right) = f\left( {x + y} \right) + f\left( x \right)f\left( y \right)$ đúng với $x;\,y$.

Từ $P(0;\,0)$ ta có $f(0)\in\{0;\,2\}$.
  1. Nếu $f(0)=2$, từ $P(x;\,0)$ có $f(x)=2\;\forall\,x\in\mathbb R$. Thử lại thấy nghiệm hàm này thoả mãn.
  2. Nếu $f(0)=0$, đặt $f(1)=k$ và từ $P(x;\,1)$ có
    \[f\left( {x + 1} \right) = \left( {2 - k} \right)f\left( x \right) + k\;\forall\,x\in\mathbb R\]
    Từ đó có
    \[\begin{array}{l}
    f\left( {x + 2} \right) &= \left( {2 - k} \right)f\left( {x + 1} \right) + k\\
    &= {\left( {2 - k} \right)^2}f\left( x \right) + 3k - {k^2}.
    \end{array}\]
    Thay $x=0$ vào để có
    \[f\left( 2 \right) = 3k - {k^2}\]
    Tức là có
    \[f\left( {x + 2} \right) = {\left( {2 - k} \right)^2}f\left( x \right) + f\left( 2 \right);\;(1).\]
    Lại từ $P(x;\,2)$ mà có
    \[f\left( {2x} \right) + f\left( x \right) + f\left( 2 \right) = f\left( {x + 2} \right) + f\left( x \right)f\left( 2 \right)\]
    Kết hợp $(1)$, đặt $3-k=l$ để có
    \[f(2x)=(3-k)f(x)=lf(x);\;(2).\]
    Từ $P(2x;\,2y)$ và $(2)$ có
    \[lf\left( {x + y} \right) + {l^2}f\left( x \right)f\left( y \right) = {l^2}f\left( {xy} \right) + lf\left( x \right) + lf\left( y \right);\;(3)\]
    • Nếu $l=0$ thì từ $(2)$ có $f(x)=0\;\forall\,x\in\mathbb R$. Thử lại thấy nghiệm hàm này thoả mãn.
    • Nếu $l=1$ tức $k=2$ thì từ $(1)$ có $f(x)=f(2)\;\forall\,x\in\mathbb R$. Tức $f(x)$ là hàm hằng và $f(x)=f(1)=2\;\forall\,x\in\mathbb R$.
      Thử lại thấy nghiệm hàm này thoả mãn.
    • Nếu $l\notin\{0;\,1\}$ thì từ $(3)$ và $P(x;\,y)$ có
      \[\begin{array}{l}
      l\left( {f\left( {xy} \right) - f\left( x \right)f\left( y \right)} \right) &= f\left( {x + y} \right) - f\left( x \right) - f\left( y \right)\\
      &= f\left( {xy} \right) - f\left( x \right)f\left( y \right)\quad\forall\,x;\,y\in\mathbb R.
      \end{array}\]
      Vậy nên có
      \[\begin{cases}
      f\left( {xy} \right) &= f\left( x \right)f\left( y \right)\\
      f\left( {x + y} \right) &= f\left( x \right) + f\left( y \right)
      \end{cases}\quad\forall\,x;\,y\in\mathbb R.\]
      Từ đây thấy $f$ cộng tính và tăng nên có nghiệm hàm $f(x)=x\;\forall\,x\in\mathbb R.$
Tóm lại, có ba nghiệm hàm thoả yêu cầu là $f_1(x)=0,\;f_2(x)=2$ và $f(x)=x$.
[RIGHT][I][B]Nguồn: MathScope.ORG[/B][/I][/RIGHT]
 
Thụy An is offline   Trả Lời Với Trích Dẫn
Old 19-10-2017, 06:05 PM   #18
babyteen9x
+Thành Viên+
 
Tham gia ngày: Jul 2010
Bài gởi: 8
Thanks: 4
Thanked 1 Time in 1 Post
Trích:
Nguyên văn bởi MATHSCOPE View Post

$\boxed{1}$ [Hà Nội] Cho $x;\, y;\, z$ là các số hữu tỉ sao cho $x+y^{2}+z^{2}$, $y+z^{2}+x^{2}$ và $z+x^{2}+y^{2}$ đều là các số nguyên. Chứng minh rằng $2x$ là số nguyên.
Ta viết $x = \dfrac{a}{m};\,y = \dfrac{b}{m};\,z = \dfrac{c}{m}$ với $a;\,b;\,c;\,m\in\mathbb Z,\;m>0$ và $\gcd(a;\,b;\,c;\,m)=1$, để có
\[\begin{array}{l}
ma + {b^2} + {c^2}\; \vdots \;{m^2}\\
mb + {c^2} + {a^2}\; \vdots \;{m^2}\\
mc + {a^2} + {b^2}\; \vdots \;{m^2}
\end{array}\]
Từ đây có $a^2+b^2;\,b^2+c^2;\,c^2+a^2$ đều là bội của $m$, do đó
\[2a^2=\left( {{c^2} + {a^2}} \right) + \left( {{a^2} + {b^2}} \right) - \left( {{b^2} + {c^2}} \right)\; \vdots \;m\]
Tương tự ta cũng có $m\mid 2b^2$ và $m\mid 2c^2$, xét hai trường hợp sau
  1. Nếu $m$ lẻ, ta có luôn $a^2;\,b^2;\,c^2$ đều là bội của $m$, mà $\gcd(a;\,b;\,c;\,m)=1$ cho nên $m=1$ tức $x\in\mathbb Z$ vì thế tất nhiên có điều cần chứng minh.
  2. Nếu $m$ chẵn, viết $m=2k$ ta có luôn $a^2;\,b^2;\,c^2$ đều là bội của $k$, mà $\gcd(a;\,b;\,c;\,2k)=1$ cho nên $k=1$ tức $m=2$ và có $2x=a\in\mathbb Z$.

[RIGHT][I][B]Nguồn: MathScope.ORG[/B][/I][/RIGHT]
 
babyteen9x is offline   Trả Lời Với Trích Dẫn
Old 19-10-2017, 06:47 PM   #19
a1npro0d9
+Thành Viên+
 
Tham gia ngày: Feb 2017
Bài gởi: 6
Thanks: 1
Thanked 1 Time in 1 Post
Trích:
Nguyên văn bởi MATHSCOPE View Post
$\boxed{40}$ [Huế] Tìm tất cả hàm số $f: \mathbb{R} \rightarrow \mathbb{R}$ thỏa mãn
\[f\left( {{x^3}} \right) + f\left( {{y^3}} \right) = (x + y)\left( {f\left( {{x^2}} \right) + f\left( {{y^2}} \right)} - f\left( {xy} \right)\right) \quad\forall \,x;\,y \in \mathbb{R}\]
Gọi $P(x;\,y)$ là mệnh đề: $f\left( {{x^3}} \right) + f\left( {{y^3}} \right) = (x + y)\left( {f\left( {{x^2}} \right) + f\left( {{y^2}} \right)} - f\left( {xy} \right)\right) $ đúng với $x;\,y$.

Từ $P(0;\,0)$ có $f(0)=0$, sau đó từ $P(x;\,0)$ có
\[f\left( {{x^3}} \right) = xf\left( {{x^2}} \right)\quad\forall \,x \in \mathbb{R}\]
Từ đây có ngay $f$ là hàm lẻ, đồng thời
\[xf\left( {{x^2}} \right) + yf\left( {{y^2}} \right) = (x + y)\left( {f\left( {{x^2}} \right) + f\left( {{y^2}} \right) - f\left( {xy} \right)} \right)\quad\forall \,x;\,y \in \mathbb{R}\]
Rút gọn lại ta được
\[yf\left( {{x^2}} \right) + xf\left( {{y^2}} \right) = \left( {x + y} \right)f\left( {xy} \right)\quad\forall \,x;\,y \in \mathbb{R};\;(1)\]
Thế $y$ bởi $-y$ vào đẳng thức trên với chú ý rằng $f$ lẻ, để có
\[\left( {y - x} \right)f\left( {xy} \right) = \left( {x - y} \right)f\left( { - xy} \right) = - yf\left( {{x^2}} \right) + xf\left( {{y^2}} \right)\quad\forall \,x;\,y \in \mathbb{R};\;(2)\]
Kết hợp $(1)$ và $(2)$ để có
\[2yf\left( {xy} \right) = xf\left( {{y^2}} \right)\quad\forall \,x;\,y \in \mathbb{R}\]
Giờ cho $y=1$ để có nghiệm hàm $f(x)=kx$ với $k$ là một hằng số thực, hàm này thoả mãn khi ta thử lại.
[RIGHT][I][B]Nguồn: MathScope.ORG[/B][/I][/RIGHT]
 
a1npro0d9 is offline   Trả Lời Với Trích Dẫn
Old 19-10-2017, 07:25 PM   #20
Cutrone
+Thành Viên+
 
Tham gia ngày: Oct 2017
Bài gởi: 8
Thanks: 2
Thanked 0 Times in 0 Posts
Trích:
Nguyên văn bởi MATHSCOPE View Post
$\boxed{27}$ [Hà Tĩnh] Cho hai đa thức bậc ba:
\[P(x)=x^3+2x^2-7x-16,\quad Q(x)=x^3+3x^2+8x-4\]
  • Chứng minh rằng mỗi đa thức đều có một nghiệm dương duy nhất.
  • Gọi các nghiệm dương của $P(x),\, Q(x)$ lần lượt là $p;\, q$. Chứng minh rằng: \[\sqrt{p}-\sqrt{q}=1.\]
  1. Có $P(0)P(3)<0$ và $Q(0)Q(1)<0$, nên theo định lý hàm liên tục thì chắc chắn $P(x)$ và $Q(x)$ có các nghiệm thực dương.

    Nếu $P(x)$ có hai nghiệm thực dương, thế thì theo Viettè tích các nghiệm của $P(x)$ là 16 nên nó có cả ba nghiệm thực dương, nhưng điều này dẫn đến vô lý vì cũng theo Viettè thì tổng các nghiệm của $P(x)$ là -2.

    Tương tự, theo Viettè tổng các nghiệm của $Q(x)$ là -3 còn tích chúng là 4, nên $Q(x)$ không thể có hai nghiệm thực dương.

    Vậy, mỗi đa thức đã cho có và có duy nhất một nghiệm thực dương.
  2. Đặt $\sqrt q =r$ thế thì do $q$ là nghiệm thực dương của $Q(x)$ và $q=r^2$, nên $r$ là nghiệm thực dương của đa thức
    \[Q\left( {{x^2}} \right) = {x^6} + 3{x^4} + 8{x^2} - 4\]
    Để ý rằng
    \[Q\left( {{x^2}} \right) =\left( {{x^3} - {x^2} + 2x + 2} \right)\left( {{x^3} + {x^2} + 2x - 2} \right)\]
    Đồng thời lại có đánh giá
    \[{x^3} - {x^2} + 2x + 2 = x{\left( {x - 1} \right)^2} + {x^2} + x + 2 > 0\;\forall\,x>0\]
    Vậy nên có ${{r^3} + {r^2} + 2r - 2}=0 $, lại xét
    \[\begin{align*}
    P\left( {{{\left( {r + 1} \right)}^2}} \right) &= {\left( {r + 1} \right)^6} + 2{\left( {r + 1} \right)^2} - 7{\left( {r + 1} \right)^2} - 16\\
    &= {r^6} + 6{r^5} + 17{r^4} + 28{r^3} + 20{r^2} - 20\\
    &= \left( {{r^3} + {r^2} + 2r - 2} \right)\left( {{r^3} + 5{r^2} + 10r + 10} \right)\\
    &= 0
    \end{align*}\]
    Vậy $(r+1)^2$ là nghiệm dương của $P(x)$, tức $p=(r+1)^2=\left(1+\sqrt q\right)^2$, và ta có điều phải chứng minh sau khi lấy căn bậc hai.

[RIGHT][I][B]Nguồn: MathScope.ORG[/B][/I][/RIGHT]
 

thay đổi nội dung bởi: Cutrone, 19-10-2017 lúc 07:27 PM
Cutrone is offline   Trả Lời Với Trích Dẫn
Old 20-10-2017, 02:15 AM   #21
a1npro0d9
+Thành Viên+
 
Tham gia ngày: Feb 2017
Bài gởi: 6
Thanks: 1
Thanked 1 Time in 1 Post
Trích:
Nguyên văn bởi MATHSCOPE View Post
$\boxed{36}$ [Hà Nội] Tìm tất cả các đa thức $P(x$) với hệ số thực sao cho:
\[P^2(x)=2P(x^2-3)+1\quad \forall x \in \mathbb{R}.\]
Nếu tồn tại đa thức $P$ thoả yêu cầu với $\deg(P)>0$, ta giả sử $P$ là đa thức nghiệm với $\deg(P)$ nhỏ nhất, từ giả thiết có
\[{P^2}\left( x \right) = 2P\left( {{x^2} - 3} \right) + 1 = 2P\left( {{{\left( { - x} \right)}^2} - 3} \right) + 1 = {P^2}\left( { - x} \right)\]
Như vậy hoặc $P(x)=P(-x)\;\forall\,x$ hoặc $P(-x)=-P(x)\;\forall\,x$.
  1. Nếu $P(x)=P(-x)\;\forall\,x$, khi đó sẽ tồn tại đa thức $p(x)$ thoả $P(x)=p\left(x^2\right)$ và có
    \[{p^2}\left( {{x^2}} \right) = 2p\left( {{{\left( {{x^2} - 3} \right)}^2}} \right) + 1\;\forall\,x\]
    Từ đó cũng có được
    \[{p^2}\left( x \right) = 2p\left( {{{\left( {x - 3} \right)}^2}} \right) + 1\;\forall\,x\]
    Đặt $p(x+3)=p_*(x)$, ta có $p_*^2(x)=2p_*\left(x^2-3\right)+1\;\forall\,x$. Như vậy $p_*$ cũng là đa thức nghiệm, nhưng nó phạm vai trò của $P$ do
    \[\deg (P) = 2\deg (p) = 2\deg \left( {{p_*}} \right).\]
    Vậy không xảy đến trường hợp này.

  2. Nếu $P(-x)=-P(x)$, ta có luôn $P(0)=0$, xét hai dãy số $\left\{x_n\right\}_{n\in\mathbb N}$ và $\left\{p_n\right\}_{n\in\mathbb N}$ với $x_0=y_0=0$ và các hệ thức truy hồi
    \[{x_{n + 1}} = x_n^2 - 3;\;{p_{n + 1}} = \frac{1}{2}\left( {p_n^2 - 1} \right)\quad\forall\,n\in\mathbb N.\]
    Ta thấy rằng
    \[\mathop {\lim }\limits_{n \to \infty } {x_n} = + \infty ;\;\mathop {\lim }\limits_{n \to \infty } {p_n} = 1 - \sqrt 2 \]
    Nhưng $p_n=P\left(x_n\right)$, điều này mâu thuẫn với
    \[\mathop {\lim }\limits_{x \to + \infty } P\left( x \right) = \infty \]
Như vậy, không tồn tại đa thức khác đa thức hằng thoả yêu cầu. Còn với trường hợp đa thức hằng, dễ dàng ta có hai nghiệm là
\[{P_1}\left( x \right) = 1 - \sqrt 2 ;\;{P_2}\left( x \right) = 1 + \sqrt 2 ;\]
[RIGHT][I][B]Nguồn: MathScope.ORG[/B][/I][/RIGHT]
 
a1npro0d9 is offline   Trả Lời Với Trích Dẫn
Old 22-10-2017, 04:27 PM   #22
hoanganhtran
Moderator
 
Tham gia ngày: Jan 2017
Bài gởi: 8
Thanks: 1
Thanked 3 Times in 3 Posts
$ HÀ NỘI $ Cho $x$ $y$ $z$ là các số hữu tỉ thỏa mãn $x+y^2+z^2$ , $y+x^2+z^2$ $z+y^2+x^2$ là các số nguyên. CMR 2x là số nguyên.
Lời gải:
Giả sử tồn tại các số hữu tỉ $x$ $y$ $z$ thỏa mãn đề bài. khi đó tồn tại các số nguyên $a$ $b$ $c$ $d$ thỏa mãn $x= \dfrac {a}{d}$, $y= \dfrac {b}{d}$, $z= \dfrac {c}{d}$ và $(a,b,c,d)=1$.
Giả thiết sẽ tương đương với $ ad+b^2+c^2 \equiv bd+a^2+c^2 \equiv cd+a^2+b^2 \equiv 0 (mod d^2)$ (1)
Giả sử d có ước nguyên tố lẻ là $p$ thì từ (1) ta có $a^2+b^2 \equiv b^2+c^2 \equiv a^2+c^2 \equiv 0 (mod p) \Rightarrow a^2 \equiv b^2 \equiv c^2 (mod p)$
Suy ra $2a^2$ $2b^2$ $2c^2$ chia hết cho $p$ mà $p$ là số nguyên tố lẻ nên $a$ $b$ $c$ đều chia hết cho p nên $(a,b,c,d) \equiv 0 (mod p)$ (mâu thuẫn với $(a,b,c,d)=1$ )
Vậy d không có ước nguyên tố hay $p$ hay d là lũy thừa của 2. Đặt $d=2^m$ ($m$ là số tự nhiên)
Giả sử $m \geq 2$ thì $a^2+b^2$ $b^2+c^2$ $ a^2+c^2$ chia hết cho 4 nên a,b,c phải cùng chẵn( do bình phương của 1 số lẻ luôn đồng dư với 1 modun 4) (mâu thuẫn với $(a,b,c,d)=1$)
Vậy $m<2$ hay d là ước của 2 suy ra $2x$ là số nguyên(đpcm).
[RIGHT][I][B]Nguồn: MathScope.ORG[/B][/I][/RIGHT]
 
hoanganhtran is offline   Trả Lời Với Trích Dẫn
The Following User Says Thank You to hoanganhtran For This Useful Post:
thepduc (22-10-2017)
Old 22-10-2017, 05:25 PM   #23
hoanganhtran
Moderator
 
Tham gia ngày: Jan 2017
Bài gởi: 8
Thanks: 1
Thanked 3 Times in 3 Posts
66 [Đắk Lắk] Tìm tất cả các bộ số nguyên $(a;b;c;d)$ thoả
$a^2+35=5^b6^c7^d$
Lời giải:
Giả sử tồn tại các số nguyên $(a,b,c,d)$ thỏa mãn đề bài. Hiển nhiên b,c,d phải là các số tự nhiên.
Nếu $b>1 \Rightarrow 5^b6^c7^d \equiv 0 (mod 5)$, lại có 35 chia hết cho 5 nên $a^2 \equiv 0 (mod 5) \Rightarrow a \equiv 0 (mod 5) \Rightarrow a^2 \equiv 0 (mod 25) \Rightarrow a^2+35 \equiv 10 (mod 25)$ mà $5^b6^c7^d \equiv 0 (mod 25)$ (mâu thuẫn) nên hoặc $b=0$ hoặc $b=1$.
CMTT ta có hoặc $d=0$ hoặc $d=1$
Trường hợp $b=0$ $d=1$ thì $ a^2+35=7.6^c$ mà $ 6^b \equiv 1 (mod 5) \Rightarrow 7.6^c \equiv 2 (mod 5) \Rightarrow a^2+35 \equiv 2 mod 5) \Rightarrow a^2 \equiv 2(mod 5)$ mà 1 số chính phương chỉa đồng dư 0,1,4 modun 5( mâu thuẫn)
Trường hợp $b=1$ $d=0$ thì $ a^2+35=5.6^c$ suy ra $a$ phải chia hết cho 5 nên $a^2 \equiv 0 (mod 25) \Rightarrow a^2+35 \equiv 10 (mod 25) \Rightarrow 5.6^c \equiv 10 (mod 25) \Rightarrow 6^c \equiv 2 (mod 5)$ ( vô lí do $6^c \equiv \pm1 (mod 7)$)
Trường hợp $ b=1$ $c=1$ suy ra $a$ chia hết cho 5 và 7. Đặt $a=35k$ với $k$ là nguyên suy ra $35k^2+1=6^c \Rightarrow 1 \equiv 6^c (mod 7) \Rightarrow c \equiv 0 (mod 2) $.
Nếu $c=0$ thì $k=0$
Nếu $c>0$ thì $35k^2+1$ là số chẵn nên $k$ lẻ suy ra $k^2 \equiv 1 (mod 8) \Rightarrow 35k^2+1 \equiv 4 (mod 8) \Rightarrow 6^c \equiv 4 (mod 8) \Rightarrow c=2 \Rightarrow k=1\Rightarrow a=35$
Trường hợp $b=c=0$ thì $a^2+35=6^c$
Do $6^c \geq 35 \Rightarrow c \geq 2 \Rightarrow a^2+35 \equiv 0 (mod 2)$ nên a lẻ suy ra $a^2+35 \equiv 4 (mod 8) \Rightarrow c=2 \Rightarrow a=1$
Kết luận no...
[RIGHT][I][B]Nguồn: MathScope.ORG[/B][/I][/RIGHT]
 
hoanganhtran is offline   Trả Lời Với Trích Dẫn
Old 22-10-2017, 06:03 PM   #24
hoanganhtran
Moderator
 
Tham gia ngày: Jan 2017
Bài gởi: 8
Thanks: 1
Thanked 3 Times in 3 Posts
4 [Đắk Lắk] Tìm số nguyên dương $n$ sao cho $(n^2+11n−4)n!+33.13^n+4$ là một số chính phương.
Lời giải:
Giả sử tồn tại n thỏa mãn đề bài. Khi đó tồn tại số tự nhiên $A$ sao cho $(n^2+11n−4)n!+33.13^n+4=A^2$
Xét $n \geq 4 \Rightarrow n! \equiv 0 (mod8)$
Nếu $n \equiv 0 (mod 2) \Rightarrow 13^n \equiv 1 (mod 8) \Rightarrow 33.13^n+4 \equiv 5 (mod 8) \Rightarrow A^2 \equiv 5 (mod 8)$ (vô lí do 1 số chính phương chỉ đồng dư 0,1,4 modun 8)
Vậy $n \geq 4$ thì n lẻ.
Ta có $ n^2+11n-4 \equiv n^2 -3n-4 \equiv (n-4)(n+1) \equiv (n+3)(n+1) (mod 7) \Rightarrow (n^2+11n-4)n! \equiv (n+3)(n+1)! (mod 7)$
Do $n \geq 4$ nên với $n$ khác 5 thì $(n+3)(n+1)! \equiv 0 (mod 7) \Rightarrow A^2 \equiv 33.13^n+4 \equiv (-2)(-1)+4=6 (mod 7)$ (do n lẻ)( vô lí do 1 số chính phương chỉ đồng dư 0,1,2,4 modun 7)
Vậy ta chỉ cần xét $n \in {1,2,3,5} $ để tìm $n$ thỏa mãn đề bài.
[RIGHT][I][B]Nguồn: MathScope.ORG[/B][/I][/RIGHT]
 

thay đổi nội dung bởi: hoanganhtran, 22-10-2017 lúc 11:05 PM
hoanganhtran is offline   Trả Lời Với Trích Dẫn
Old 22-10-2017, 06:33 PM   #25
hoanganhtran
Moderator
 
Tham gia ngày: Jan 2017
Bài gởi: 8
Thanks: 1
Thanked 3 Times in 3 Posts
7 [Hoà Bình] Cho $a;b;c$ là 3 số nguyên thỏa mãn
$$a+b+c=a^2(c−b)+b^2.(a−c)+c^2(b−a)$$
Chứng minh rằng $a+b+ca+b+c$ chia hết cho 27.
Lời giải:
Giả sử tồn tại các số nguyên $a;b;c$ thỏa mãn đề bài.
Ta có $$a+b+c=a^2(c−b)+b^2.(a−c)+c^2(b−a)=(a-b)(b-c)(c-a)$$
Nếu $a;b;c$ lập thành hệ thặng dư đầy đủ modun 3 thì $a+b+c \equiv 0 (mod 3) \Rightarrow (a-b)(b-c)(c-a) \equiv 0 (mod 3)$ nên tồn tại 2 trong 3 số đồng dư với nhau trong modun 3 nên $(a;b;c)$ không lập thành hệ thặng dư đầy đủ modun 3(mâu thuẫn)
Nếu $(a;b;c)$ không lập thành hệ thặng dư đầy đủ modun 3 tức là trong 3 số a;b;c có 2 số đồng dư với nhau theo modun 3 nên$$(a-b)(b-c)(c-a) \equiv 0 (mod 3) \Rightarrow a+b+c \equiv 0 (mod 3)$$.
Không mất tính tổng quát, giả sử $$a \equiv b (mod 3) \Rightarrow 2a+c \equiv 0 (mod 3) \Rightarrow a \equiv c (mod 3) \Rightarrow a-b \equiv b-c \equiv c-a \equiv 0 (mod 3) \Rightarrow (a-b)(b-c)(c-a) \equiv 0 (mod 27)$$
Suy ra $a+b+c$ chia hết cho 27(đpcm)
[RIGHT][I][B]Nguồn: MathScope.ORG[/B][/I][/RIGHT]
 
hoanganhtran is offline   Trả Lời Với Trích Dẫn
Old 22-10-2017, 08:56 PM   #26
thepduc
+Thành Viên+
 
Tham gia ngày: Jul 2017
Bài gởi: 5
Thanks: 1
Thanked 1 Time in 1 Post
Trích:
Nguyên văn bởi hoanganhtran View Post
Nếu $n \geq 3 \Rightarrow n! \equiv 0 (mod 3) \Rightarrow (n^2+11n−4)n!+33.13^n+4 \equiv 2 (mod 3)$
Mình nghĩ bạn Hoàng Anh nhầm chỗ này, phải là
$$(n^2+11n−4)n!+33.13^n+4 \equiv 1 \pmod 3 $$
[RIGHT][I][B]Nguồn: MathScope.ORG[/B][/I][/RIGHT]
 
thepduc is offline   Trả Lời Với Trích Dẫn
The Following User Says Thank You to thepduc For This Useful Post:
hoanganhtran (22-10-2017)
Old 22-10-2017, 09:17 PM   #27
Thụy An
+Thành Viên+

 
Tham gia ngày: Oct 2017
Bài gởi: 93
Thanks: 1
Thanked 68 Times in 45 Posts
Trích:
Nguyên văn bởi MATHSCOPE View Post
$\boxed{16}$ [Chuyên KHTN Hà Nội] Cho dãy số $\left(a_n\right)$ xác định bởi công thức sau: \[a_0=1;\,a_1=4;\,a_{n+2}=2a_{n+1}+3a_n\quad\forall \,n \in\mathbb N.\]
Chứng minh rằng trong dãy số trên không có số nào là bội của 2017.
Giải phương trình đặc trưng $x^2=2x+3$ để có công thức truy hồi của dãy là
\[{a_n} = \frac{{{{5.3}^n} - {{\left( { - 1} \right)}^n}}}{4}\quad\forall\,n\in\mathbb N.\]
Giả sử tồn tại số tự nhiên $n$ để $2017\mid a_n$, lúc đó có đồng dư
\[{5.3^n} \equiv {\left( { - 1} \right)^n}\pmod{2017};\;(*)\]
Để ý rằng $\left( {\dfrac{{ - 1}}{{2017}}} \right) = {\left( { - 1} \right)^{\frac{{2017 - 1}}{2}}} = 1$ , và theo luật thuận nghịch thì
\[\left( {\frac{3}{{2017}}} \right)\left( {\frac{{2017}}{3}} \right) = {\left( { - 1} \right)^{\frac{{\left( {3 - 1} \right)\left( {2017 - 1} \right)}}{4}}} = 1\]
Trong khi $2017^{\frac{3-1}{2}}\equiv 1\pmod 3$, cho nên chứng tỏ $3$ là thặng dư bậc hai theo mod $2017$.

Những điều đó kết hợp với $(*)$ cho ta $5$ là thặng dư bậc hai mod $2017$; $(1)$, đồng thời
\[\left( {\frac{5}{{2017}}} \right)\left( {\frac{{2017}}{5}} \right) = {\left( { - 1} \right)^{\frac{{\left( {5 - 1} \right)\left( {2017 - 1} \right)}}{4}}} = 1\]
Có điều, $2017^{\frac{5-1}{2}}\equiv -1\pmod 5$ nên
\[\left( {\frac{5}{{2017}}} \right) = \left( {\frac{{2017}}{5}} \right) = - 1;\;(2).\]
Từ mâu thuẫn của $(1)$ và $(2)$, cho ta điều cần chứng minh.
[RIGHT][I][B]Nguồn: MathScope.ORG[/B][/I][/RIGHT]
 
Thụy An is offline   Trả Lời Với Trích Dẫn
Old 23-10-2017, 04:51 PM   #28
hoanganhtran
Moderator
 
Tham gia ngày: Jan 2017
Bài gởi: 8
Thanks: 1
Thanked 3 Times in 3 Posts
20 [Hà Tĩnh] Tìm tất cả các cặp số nguyên $(a;b)$ sao cho với mọi số nguyên dương $n$, ta có $n$ chia hết cho $a^n+b^{n+1}$.
Lời giải:
Giả sử tồn tại bộ số nguyên dương $(a,b)$ thỏa mãn đề bài.
Ta thấy nếu một trong 2 số $a;b$ bằng 0 thì số còn lại là $ \in1$.
Xét trường hợp cả a và b đều khác 0.
Theo giả thiết, ta chọn $p$ là số nguyên tố và $p> |a|+|b^2|$ thì $p>a;b \Rightarrow (a,p)=(b,p)=1$ và $a^p+b^{p+1}$ là ước của $p \Rightarrow a^p+b^{p+1} \in {1;-1;p;-p}$
Do $(a,p)=(b,p)=1 \Rightarrow a^n+b^{n+1} \equiv a+b^2 (mod p) \Rightarrow (a^n+b^{n+1},p)=1$ (vì $a+b^2 <p$)
Suy ra $a^n+b^{n+1} \pm 1 $ nên $a;b$ khác tính chẵn lẻ hay $a^n+b^{n+1} \equiv 1 (mod 2) \forall n \in N$
Chọn $n=2^k$ với $k$ là số tự nhiên. Theo giả thiết ta có $a^{2^k}+b^{2^k+1}$ là ước của $2^k$ mà $a;b$ khác tính chẵn lẻ nên $a^{2^k}+b^{2^k+1}= \pm1 \forall k \in N*$
Suy ra tồn tại dãy số tự nhiên vô hạn S={$ k_1;k_2;... $}
sao cho $a^n+b^{n+1} =1 \forall n \in S$ hoặc $a^n+b^{n+1} =-1 \forall n \in S$
Trường hợp 1: $a^n+b^{n+1} =1 \forall n \in S$ hay $a^{2^{k_i}}+b^{2^{k_i}+1} =1 \forall n \in S$
Nếu $a$ lẻ thì $V_2(a^{2^(k_i)}-1)=V_2(b^{2^(k_i)+1} \Rightarrow V_2(a^2-1)+V_2(2^{k_i})-1=(2^{k_i}+1).V_2(b) \Rightarrow V_2(a^2-1)>2^{k_i}-k_i$, cho i chạy đến vô cùng thì $V_2(a^2-1)$ cũng tiến tới vô cùng nên $a= \pm 1$
Nếu $a$ chẵn ta có $V_2(b^{2^(k_i)+1}-1)=V_2(b-1)=V_2(a^{2^(k_i)})$ cho i chạy tới vô cùng( vô lí)
Trường hợp 2 làm tương tự.
Vậy 1 trong 2 số luôn có 1 số bằng $\pm 1$
Nếu $a=1$ thì $b^{n+1}+1$ là ước của n với mọi n. Chọn n bằng 1 thì $b=0$
Nếu $a=-1$ thì chọn n=1 thì b=0
Nếu $b=1$ thì chọn n=1 và n=2 suy ra a=0
Nếu $b=-1$ thì chọn n=1 và n=2 suy ra a=0.
[RIGHT][I][B]Nguồn: MathScope.ORG[/B][/I][/RIGHT]
 
hoanganhtran is offline   Trả Lời Với Trích Dẫn
The Following User Says Thank You to hoanganhtran For This Useful Post:
MATHSCOPE (23-10-2017)
Old 24-10-2017, 08:36 AM   #29
Le khanhsy
Super Moderator
 
Tham gia ngày: Oct 2017
Bài gởi: 48
Thanks: 52
Thanked 57 Times in 30 Posts
Trích:
Nguyên văn bởi MATHSCOPE View Post
$\boxed{1}$ [Quảng Ninh] Cho ba số thực dương $a;\, b;\, c$ có tổng bằng 3. Chứng minh rằng:
\[\dfrac{{{a^2}}}{{2a + 1}} + \dfrac{{{b^2}}}{{2b + 1}} + \dfrac{{{c^2}}}{{2c + 1}} \leqslant \dfrac{{{a^2} + {b^2} + {c^2}}}{{\sqrt {{a^2} + {b^2} + {c^2} + 6} }}\]
.
Để ý rằng
$$\dfrac{x^2}{2x+1}=\dfrac{x}{2}-\dfrac{x}{2(2x+1)}.$$
Do đó bài toán viết lại như sau
$$\dfrac{a^2+b^2+c^2}{\sqrt{a^2+b^2+c^2+6}}+\dfrac {a}{2(2a+1)}+\dfrac{b}{2(2b+1)}+\dfrac{c}{2(2c+1)} \ge \dfrac{3}{2}.$$
Áp dụng Cauchy-Schwarz, ta có
$$\dfrac{a}{2(2a+1)}+\dfrac{b}{2(2b+1)}+\dfrac{c}{ 2(2c+1)}\ge \dfrac{(a+b+c)^2}{4(a^2+b^2+c^2)+2(a+b+c)}=\dfrac{ 9}{4(a^2+b^2+c^2)+6}.$$
Từ đây để bất đẳng thức đúng chúng ta chỉ cần chứng minh với $3\le t \le 9$ thì
$$\dfrac{t}{\sqrt{t+6}}+\dfrac{9}{4t+6}\ge \dfrac{3}{2},$$
hay
$$2t+3\ge 3\sqrt{t+6},$$
rút gọn tiếp tục ta được
$$(t-3)(4t+15)\ge 0.$$
Hoàn tất chứng minh. Đẳng thức xảy ra khi và chỉ khi $a=b=c=1$.
[RIGHT][I][B]Nguồn: MathScope.ORG[/B][/I][/RIGHT]
 

thay đổi nội dung bởi: Le khanhsy, 24-10-2017 lúc 09:09 AM Lý do: Tự động gộp bài
Le khanhsy is offline   Trả Lời Với Trích Dẫn
The Following 2 Users Say Thank You to Le khanhsy For This Useful Post:
2M (24-10-2017), MATHSCOPE (24-10-2017)
Old 24-10-2017, 08:36 AM   #30
Le khanhsy
Super Moderator
 
Tham gia ngày: Oct 2017
Bài gởi: 48
Thanks: 52
Thanked 57 Times in 30 Posts
Trích:
Nguyên văn bởi MATHSCOPE View Post
$\boxed{7}$ [Quảng Trị] Cho các số thực dương $x;\,y;\,z$ thay đổi và thoả mãn $xyz=1$. Tìm giá trị lớn nhất của biểu thức
\[A = \left( {x + y + z} \right)\left( {6 - \frac{x}{y} - \frac{y}{z} - \frac{z}{x}} \right)\].
Trước tiên chúng ta có bổ đề quen thuộc sau. Với các số thực dương $x,y,z$ thỏa $xyz=1$ thì
$$\dfrac{x}{y}+\dfrac{y}{z}+\dfrac{z}{x}\ge x+y+z .$$
Áp dụng bổ đề trên thì
$$A\le (x+y+z)(6-x-y-z)= 9-(x+y+z-3)^2 \le 9$$
Hoàn tất chứng minh. Đẳng thức xảy ra khi và chỉ khi $x=y=z=1$.
Bài toán trên có thể mở rộng hơn như sau
[Mở rộng] Cho các số thực dương $x;\,y;\,z$ thay đổi và thoả mãn $xyz=1$. Tìm giá trị lớn nhất của biểu thức
\[A = \sqrt{3(x^2+y^2+z^2)}\left( {6 - \frac{x}{y} - \frac{y}{z} - \frac{z}{x}} \right)\]
[RIGHT][I][B]Nguồn: MathScope.ORG[/B][/I][/RIGHT]
 
Le khanhsy is offline   Trả Lời Với Trích Dẫn
The Following 2 Users Say Thank You to Le khanhsy For This Useful Post:
2M (24-10-2017), MATHSCOPE (24-10-2017)
Trả lời Gởi Ðề Tài Mới

Bookmarks


Quuyền Hạn Của Bạn
You may not post new threads
You may not post replies
You may not post attachments
You may not edit your posts

BB code is Mở
Smilies đang Mở
[IMG] đang Mở
HTML đang Tắt

Chuyển đến


Múi giờ GMT. Hiện tại là 02:49 PM.


Powered by: vBulletin Copyright ©2000-2024, Jelsoft Enterprises Ltd.
Inactive Reminders By mathscope.org
[page compression: 116.43 k/132.18 k (11.92%)]